Inscription / Connexion Nouveau Sujet
Niveau Maths sup
Partager :

Convergence

Posté par
Jaina
25-02-06 à 15:04

Bonjour,

Je n'arrive pas à répondre à cette question, pourriez vous m'aider ?

On a :

An(z)=\int{e^{zt}\sin^{2n}t dt}

(L'intégrale est de 0 à \Pi).

On veut montrer que la suite

n\rightarrow An(z)/An(0)

converge vers exp(z\Pi/2).

Merci.

Posté par
Ksilver
re : Convergence 25-02-06 à 15:22

Salut !

hum ?  soit j'ai rater quel que chose, soit An(z)/An(0) n'est pas definit non ?

Posté par
Jaina
re : Convergence 25-02-06 à 15:27

Pourquoi ?

An(0)=\int{sin^{2n}t dt \neq {0} !

.

Posté par
Jaina
re : Convergence 25-02-06 à 18:46

Personne ne peut m'aider ?

J'ai essayé de montrer que le module de ( An(z)/An(0) - exp(z\Pi/2 )
était plus petit qu'une suite convergent vers 0, mais je n'y arrive pas.
(La meilleure majoration que j'ai trouvé est que ce module est inférieur à 2 ... )

Merci.

Posté par
kaiser Moderateur
re : Convergence 26-02-06 à 14:24

Bonjour Jaina

Je crois avoir trouvé comment résoudre ce problème et j'espère ne pas avoir fait d'erreur de raisonnement.
Mais d'abord, je voudrais dire pourquoi, finalement, ce résultat n'est pas si étonnant que ça.

En fait, sur \Large{[0,\pi]} le sinus est positif et toujours strictement inférieur à 1, sauf en \Large{\frac{\pi}{2}} où il vaut 1.
Ainsi, pour tout t différent \Large{\frac{\pi}{2}}, la quantité \Large{e^{zt}sin^{2n}(t)} tend vers 0. Par ailleurs, la suite \large{\frac{A_{n}(z)}{A_{n}(0)}} peut être vue comme un barycentre et lorsque n tend vers l'infini la "masse est concentrée" en \Large{\frac{\pi}{2}} d'où la résultat attendu.
Maintenant, après cette digression, intéressons nous au problème initial. Pour cela, on va essayer de simplifier le problème en généralisant (je sais, ça paraît bizarre, vu comme c'est dit).
Plus précisément, on va montrer que si f est une fonction continue de \Large{[0,\pi]} à valeurs complexes, alors la quantité \Large{\frac{\bigint_{0}^{\pi}f(t)sin^{2n}(t)dt}{\bigint_{0}^{\pi}sin^{2n}(t)dt}} converge vers \Large{f(\frac{\pi}{2})}.
On remarque que ce problème est linéaire par rapport à f. On peut donc se limiter au cas des fonctions réelles. En effet, pour une fonction à valeurs complexes, on peut regarder ses parties réelle et imaginaire.


Passons à la preuve proprement dite :

Considérons \Large{\epsilon >0} quelconque.
En utilisant la continuité de f en \Large{\frac{\pi}{2}}, on sait qu'il existe un réel \Large{\delta} strictement positif tel que pour tout t vérifiant \Large{|t-\frac{\pi}{2}|\leq \delta }, verifie également \Large{|f(t)-f(\frac{\pi}{2})|\leq \frac{\epsilon}{2}}. Quitte à diminuer \Large{\delta}, on peut toujours supposer que \Large{\delta < \frac{\pi}{4}}.

\Large{\bigint_{0}^{\pi}f(t)sin^{2n}(t)dt=\bigint_{0}^{\frac{\pi}{2}-\delta}f(t)sin^{2n}(t)dt+\bigint_{\frac{\pi}{2}-\delta}^{\frac{\pi}{2}+\delta}f(t)sin^{2n}(t)dt+\bigint_{\frac{\pi}{2}+\delta}^{\pi}f(t)sin^{2n}(t)dt}

Or \Large{|\bigint_{\frac{\pi}{2}+\delta}^{\pi}f(t)sin^{2n}(t)dt|\leq ||f||_{\infty}\bigint_{\frac{\pi}{2}+\delta}^{\pi}sin^{2n}(t)dt\leq ||f||_{\infty}\bigint_{\frac{\pi}{2}+\delta}^{\pi}sin^{2n}(\frac{\pi}{2}+\delta)dt=(\frac{\pi}{2}-\delta) ||f||_{\infty}sin^{2n}(\frac{\pi}{2}+\delta)}.

Par ailleurs, on a :
\Large{\bigint_{0}^{\pi}sin^{2n}(t)dt\geq \bigint_{\frac{\pi}{2}}^{\frac{\pi}{2}+2\delta}sin^{2n}(t)dt \geq \bigint_{\frac{\pi}{2}}^{\frac{\pi}{2}+2\delta}sin^{2n}(\frac{\pi}{2}+2\delta)dt=2\delta sin^{2n}(\frac{\pi}{2}+2\delta)}.

Ainsi, on a :

\Large{\frac{\bigint_{\frac{\pi}{2}+\delta}^{\pi}f(t)sin^{2n}(t)dt}{\bigint_{0}^{\pi}sin^{2n}(t)}\leq \frac{(\frac{\pi}{2}-\delta) ||f||_{\infty}sin^{2n}(\frac{\pi}{2}+\delta)}{2\delta sin^{2n}(\frac{\pi}{2}+2\delta)}=\frac{(\frac{\pi}{2}-\delta) ||f||_{\infty}}{2\delta}\(\frac{sin(\frac{\pi}{2}+\delta)}{sin(\frac{\pi}{2}+2\delta)}\)^{2n}}.

Or \Large{\frac{sin(\frac{\pi}{2}+\delta)}{sin(\frac{\pi}{2}+2\delta)}<1}, donc la quantité précédente tend vers 0 lorsque n tend vers l'infini.
En particulier, il existe un entier \Large{n_{0}} tel que pour tout entier n supérieur à \Large{n_{0}}, cette quantité est en valeur absolue, majorée par \Large{\frac{\epsilon}{4}}.

De même, il existe en un entier n_{1} et que pour tout entier qui lui est supérieur, on a \|\Large{\frac{\bigint_{0}^{\frac{\pi}{2}-\delta}f(t)sin^{2n}(t)dt}{\bigint_{0}^{\pi}sin^{2n}(t)dt}\|\leq \Large{\frac{\epsilon}{4}}
Ainsi, pour tout entier n supérieur à \Large{N=\max(n_{0},n_{1})}, ces deux inégalités sont vérifiées.

\Large{\bigint_{\frac{\pi}{2}-\delta}^{\frac{\pi}{2}+\delta}f(t)sin^{2n}(t)dt=\bigint_{\frac{\pi}{2}-\delta}^{\frac{\pi}{2}+\delta}(f(t)-f(\frac{\pi}{2}))sin^{2n}(t)dt+f(\frac{\pi}{2})\bigint_{\frac{\pi}{2}-\delta}^{\frac{\pi}{2}+\delta}sin^{2n}(t)dt}.

Or par définition de \Large{\delta}, on a :

\Large{\|\bigint_{\frac{\pi}{2}-\delta}^{\frac{\pi}{2}+\delta}(f(t)-f(\frac{\pi}{2}))sin^{2n}(t)dt\|\leq \bigint_{\frac{\pi}{2}-\delta}^{\frac{\pi}{2}+\delta}\frac{\epsilon}{2} sin^{2n}(t)dt}

On en déduit que \Large{\|\frac{\bigint_{\frac{\pi}{2}-\delta}^{\frac{\pi}{2}+\delta}f(t)sin^{2n}(t)dt}{\bigint_{0}^{\pi}sin^{2n}(t)dt}-f(\frac{\pi}{2})\|\leq \frac{\epsilon}{2} \frac{\bigint_{\frac{\pi}{2}-\delta}^{\frac{\pi}{2}+\delta}sin^{2n}(t)dt}{\bigint_{0}^{\pi}sin^{2n}(t)dt} \leq \frac{\epsilon}{2} \frac{\bigint_{0}^{\pi}sin^{2n}(t)dt}{\bigint_{0}^{\pi}sin^{2n}(t)dt}=\frac{\epsilon}{2}}.

En remettant tous les morceaux ensembles, on a pour tout n supérieur à N :

\Large{\|\frac{\bigint_{\frac{\pi}{2}+\delta}^{\pi}f(t)sin^{2n}(t)dt}{\bigint_{0}^{\pi}sin^{2n}(t)}-f(\frac{\pi}{2})\|\leq \frac{\epsilon}{4}+\frac{\epsilon}{4}+\frac{\epsilon}{2}=\epsilon}
Ceci montre bien que la suite \Large{\(\frac{\bigint_{\frac{\pi}{2}+\delta}^{\pi}f(t)sin^{2n}(t)dt}{\bigint_{0}^{\pi}sin^{2n}(t)}\)} converge vers \Large{f(\frac{\pi}{2})}

Maintenant, soit z un complexe quelconque t et posons \Large{f(t)=e^{zt}}.
Alors d'après ce qui précède, on a bien

\large{\(\frac{A_{n}(z)}{A_{n}(0)}\)} qui converge vers \Large{f(\frac{\pi}{2}})=e^{z\frac{\pi}{2}}}.

Ouf !
Encore une fois, j'espère ne pas m'être trompé.

Kaiser

Posté par
kaiser Moderateur
re : Convergence 26-02-06 à 15:06

Il faut bien sûr lire :

\Large{\|\frac{\bigint_{0}^{\pi}f(t)sin^{2n}(t)dt}{\bigint_{0}^{\pi}sin^{2n}(t)}-f(\frac{\pi}{2})\|\leq \frac{\epsilon}{4}+\frac{\epsilon}{4}+\frac{\epsilon}{2}=\epsilon}

Posté par
Youpi
re : Convergence 26-02-06 à 15:12

Belle démo Kaiser ..quel courage !

Posté par
kaiser Moderateur
re : Convergence 26-02-06 à 15:17

Bonjour Youpi
Merci bien !
P.S : J'en déduis que tu n'y a pas trouvé d'erreurs. ça me rassure.

Posté par
Jaina
re : Convergence 26-02-06 à 16:43

Merci beaucoup !!!

Je n'avais pas du tout pensé à utiliser le fait que la fonction f est continue !!

Après j'ai réussi à retrouver le résultat, encore merci !

Je bloque sur une autre question , si tu pourrais encore m'aider (si cela ne te dérange pas)

Il faut vérifier que pour n>0, on a

(1+ (z/2n)2 )Hn(z)=(1-1/2n)Hn-1(z)

J'ai essayé de le montrer par récurrence, mais je n'arrive même pas à faire l'initaition.
Peut-être faut il trouver le résultat directement, mais dans ce cas, les intégrales à calculer sont trop compliquées... Je ne m'en sort pas.

Posté par
kaiser Moderateur
re : Convergence 26-02-06 à 16:48

Mais, je t'en prie !

A-t-on \Large{H_{n}(z)=\frac{A_{n}(z)}{A_{n}(0)}} ?

Kaiser

Posté par
Jaina
re : Convergence 26-02-06 à 17:33

Non, Hn(z)=An(z)

Posté par
kaiser Moderateur
re : Convergence 26-02-06 à 17:44

\Large{H_{n}(z)-H_{n-1}(z)=\bigint_{0}^{\pi}e^{zt}sin^{2n-2}(sin^{2}(t)-1)dt=-\bigint_{0}^{\pi}e^{zt}sin^{2n-2}cos^{2}(t)dt}

À ce niveau, essaie de faire une intégration par partie en choisissant de dériver t\Large{cos(t)e^{zt}}

Kaiser

Posté par
Jaina
re : Convergence 26-02-06 à 18:35

J'ai fait l'intégration par parties, je trouve :

H_{n}(z)-H_{n-1}(z)=\int_0 ^{\Pi} e^{zt}(cos(t)/z-sin(t))(sin^{2n-1}(t)/2n-1)

Mais la je suis coincé. Je pense que je dois faire apparître 1 - 1/2n avec le 2n-1, mais je ne vois pas pour le reste.

Est-ce que c'est bon ? Comment faire pour la suite ?

Posté par
Jaina
re : Convergence 26-02-06 à 18:49

J'ai trouvé !

Il faut faire une autre intégration par partie, et je ne me retrouve plus qu'avec des Hn(z).

Maintenant, il ne me reste plus qu'à arranger l'expression pour retrouver celle donnée !

Merci beaucoup !!!

Posté par
Jaina
re : Convergence 26-02-06 à 18:58

Il me reste une dernière question, si tu pouvais encore me donner une piste...

Il faut en déduire:

(H_n(z)/H_0(z)) (1+ (z/2k)2)=H_n(0)/H_0(0)

Je vois bien qu'en faisant le produit des Hn(z)/H_{n-1}(z)

on se rapproche du résultat voulu, mais j'ai

(1-1/2k) qui me gêne, et je n'arive pas à retrouver l'expression voulue.

Merci encore pour ton aide (comment as tu fais pour voir tout de suite qu'il fallait faire une intégration par partie pour la question précédente ? Je n'y aurais jamais pensé je crois ...)

Posté par
kaiser Moderateur
re : Convergence 26-02-06 à 19:15

Normalement, tu as dû trouvé :
\Large{\frac{H_{n}}{H_{0}(z)}\bigprod_{k=1}^{n}(1+\(\frac{z}{2k}\)^{2})=\bigprod_{k=1}^{n}(1-\frac{1}{2k})}.

Que remarques-tu lorsque tu remplace z par 0 ?

Kaiser

Posté par
kaiser Moderateur
re : Convergence 26-02-06 à 19:19

J'oubliais : tu me demandais comment j'avais "tout de suite" pensé à l'intégration par parties.
En fait, au début, j'étais parti sur cette première idée pour résoudre le problème que tu as posté tout au début de ce topic et comme j'avais déjà fait les calculs ...

Posté par
Jaina
re : Convergence 26-02-06 à 20:03

Oh merci encore !!!

Je pense que cela va aller maintenant.

Merci beaucoup de m'avoir aidé !!



Posté par
kaiser Moderateur
re : Convergence 26-02-06 à 20:04

Mais je t'en prie !



Vous devez être membre accéder à ce service...

Pas encore inscrit ?

1 compte par personne, multi-compte interdit !

Ou identifiez-vous :


Rester sur la page

Inscription gratuite

Fiches en rapport

parmi 1675 fiches de maths

Désolé, votre version d'Internet Explorer est plus que périmée ! Merci de le mettre à jour ou de télécharger Firefox ou Google Chrome pour utiliser le site. Votre ordinateur vous remerciera !